Bạn chưa đăng nhập. Vui lòng đăng nhập để hỏi bài

Những câu hỏi liên quan
Quỳnh Hương
Xem chi tiết
Đen đủi mất cái nik
Xem chi tiết
Nguyễn Tiến Đức
10 tháng 9 2018 lúc 19:41

tự ra câu hởi tự trả lời à bạn

Đen đủi mất cái nik
10 tháng 9 2018 lúc 19:44

tại tui trả lời bài này cho 1 bạn ở trên facebook nên phải chụp màn hình lại nên làm v á

trang huyen
Xem chi tiết
Thắng Nguyễn
5 tháng 4 2017 lúc 21:29

Bài 3: y hệt bài mình đã từng đăng Câu hỏi của Thắng Nguyễn - Toán lớp 9 - Học toán với OnlineMath- trước mình có ghi lời giải mà lâu ko xem giờ quên r` :)

Hà Trang
5 tháng 4 2017 lúc 23:09

1) Đặt n+1 = k^2

2n + 1 = m^2

Vì 2n + 1 là số lẻ => m^2 là số lẻ => m lẻ 

Đặt m = 2t+1

=> 2n+1 = m^2 = (2t+1)^2

=> 2n+1 = 41^2 + 4t + 1

=> n = 2t(t+1)

=> n là số chẵn

=> n+1 là số lẻ

=> k lẻ 

+) Vì k^2 = n+1

=> n = (k-1)(k+1)

Vì k -1 và k+1 là 2 số chẵn liên tiếp

=> (k+1)(k-1) chia hết cho * 

=> n chia hết cho 8

+) k^2 + m^2 = 3a + 2

=> k^2 và m^2 chia 3 dư 1

=> m^2 - k^2 chia hết cho 3

m^2 - k^2 = a

=> a chia hết cho 3

Mà 3 và 8 là 2 số nguyên tố cùng nhau

=> a chia hết cho 24

Hà Trang
5 tháng 4 2017 lúc 23:10

ấy nhầm, là n chứ không phải a nha :))

Thanh Xuân
Xem chi tiết
Thanh Xuân
Xem chi tiết
Trà My
19 tháng 7 2016 lúc 17:38

bài 28

\(P=\frac{\left[a^2-\left(b+c\right)^2\right]\left(a+b-c\right)}{\left(a+b+c\right)\left[\left(a-c\right)^2-b^2\right]}\)

=>\(P=\frac{\left(a-b-c\right)\left(a+b+c\right)\left(a+b-c\right)}{\left(a+b+c\right)\left(a-c-b\right)\left(a-c+b\right)}\)

=>\(P=1\)

Nguyễn Lương Hà
19 tháng 7 2016 lúc 18:37

Bài 30 phải là xy+y+x=3.

Ta có: xy+y+x=3 => (x+1)(y+1)=4(1)

            yz+y+z=8 => (y+1)(z+1)=9(2)

           zx+x+z=15 => (x+1)(z+1)=16(3)

Nhân (1), (2) và (3) theo vế, ta có:

           [(x+1)(y+1)(z+1)]2=576

     =>  (x+1)(y+1)(z+1)=24(I) hoặc (x+1)(y+1)(z+1)=-24(II)

Lần lượt thay (1),(2),(3) vào (I),(II), tính x,y,z.

Kết quả: P=43/6 hoặc P=-79/6

Thanh Xuân
Xem chi tiết
Đặng Tuấn Anh
Xem chi tiết
Lưu Đức Mạnh
26 tháng 3 2018 lúc 23:50

Đầu tiên ta sẽ chứng minh \(\frac{x^2}{a}+\frac{y^2}{b}\ge\frac{\left(x+y\right)^2}{a+b}\left(1\right)\)

\(\Leftrightarrow x^2b\left(a+b\right)+y^2a\left(a+b\right)\ge ab\left(x+y\right)^2\)

\(\Leftrightarrow\left(bx-ay\right)^2\ge0\left(LĐ\right)\)

Dấu "=" xảy ra khi \(\frac{x}{a}=\frac{y}{b}\)

Vậy BĐT (1) đã được chứng minh

Với 6 số x,y,z,a,b,c >0 ta sẽ áp dụng BĐT (1) hai lần:

\(\frac{x^2}{a}+\frac{y^2}{b}+\frac{z^2}{c}\ge\frac{\left(x+y\right)^2}{a+b}+\frac{z^2}{c}\ge\frac{\left(x+y+z\right)^2}{a+b+c}\left(đpcm\right)\)

Nguyễn Minh Đăng
22 tháng 7 2020 lúc 9:47

Bài làm:

Áp dụng Cauchy Schwars ta có:

\(\frac{x^2}{a}+\frac{y^2}{b}+\frac{z^2}{c}\ge\frac{\left(x+y+z\right)^2}{a+b+c}\)

Dấu "=" xảy ra khi: \(\frac{x}{a}=\frac{y}{b}=\frac{z}{c}\)

Khách vãng lai đã xóa
ミ★Ƙαї★彡
22 tháng 7 2020 lúc 10:24

Áp dụng BĐT Svac - xơ ta có :

\(\frac{x^2}{a}+\frac{y^2}{b}+\frac{z^2}{c}\ge\frac{\left(x+y+z\right)^2}{a+b+c}\)

Dấu ''='' xảy ra <=> \(\frac{x}{a}=\frac{y}{b}=\frac{z}{c}\)

Khách vãng lai đã xóa
Tuấn Minh Nguyễn
Xem chi tiết
Phùng Minh Quân
6 tháng 11 2018 lúc 21:00

\(ab+bc+ca\le a^2+b^2+c^2\le\frac{\left(a+b+c\right)^2}{3}\) ( bđt phụ + Cauchy-Schwarz dạng Engel ) 

Dấu "=" xảy ra \(\Leftrightarrow\)\(a=b=c\)

CM bđt phụ : \(x^2+y^2+z^2\ge xy+yz+zx\)

\(\Leftrightarrow\)\(2x^2+2y^2+2z^2\ge2xy+2yz+2zx\)

\(\Leftrightarrow\)\(2x^2+2y^2+2z^2-2xy-2yz-2zx\ge0\)

\(\Leftrightarrow\)\(\left(x^2-2xy+y^2\right)+\left(y^2-2yz+z^2\right)+\left(z^2-2zx+x^2\right)\ge0\)

\(\Leftrightarrow\)\(\left(x-y\right)^2+\left(y-z\right)^2+\left(z-x\right)^2\ge0\) ( luôn đúng ) 

Dấu "=" xảy ra \(\Leftrightarrow\)\(x=y=z\)

Chúc bạn học tốt ~ 

Yoona
Xem chi tiết
Phương An
25 tháng 1 2017 lúc 16:22

\(\frac{1}{x}+\frac{1}{y}+\frac{1}{z}=0\)

\(\frac{yz}{xyz}+\frac{xz}{xyz}+\frac{xy}{xyz}=0\)

\(\frac{yz+xz+xy}{xyz}=0\)

yz + xz + xy = 0

\(\left(x+y+z\right)^2=x^2+y^2+z^2+2xy+2xz+2yz=x^2+y^2+z^2+2\times\left(xy+xz+yz\right)=x^2+y^2+z^2+2\times0=x^2+y^2+z^2\left(\text{đ}pcm\right)\)

Nguyen Bao Linh
25 tháng 1 2017 lúc 17:40

a) Từ giả thiết suy ra: xy + yz + zx = 0

Do đó:

\(\left(x+y+z\right)^2=x^2+y^2+z^2+2\left(xy+yz+zx\right)=x^2+y^2+z^2\)

b) Đặt \(\frac{1}{a-b}=x\); \(\frac{1}{b-c}=y\); \(\frac{1}{c-a}=z\)

Ta có: \(\frac{1}{x}+\frac{1}{y}+\frac{1}{z}=a-b+b-c+c-a=0\)

Theo câu a ta có: \(x^2+y^2+z^2=\left(x+y+z\right)^2\)

Suy ra điều phải chứng minh

Kuro Kazuya
25 tháng 1 2017 lúc 17:59

a)

\(\frac{1}{x}+\frac{1}{y}+\frac{1}{z}=0\)

\(\Rightarrow\frac{xy+yz+xz}{xyz}=0\)

\(\Rightarrow xy+yz+xz=0\)

\(x^2+y^2+z^2=\left(x+y+z\right)^2\)

\(\Rightarrow x^2+y^2+z^2=x^2+y^2+z^2+2xy+2yz+2xz\)

\(\Rightarrow x^2+y^2+z^2=x^2+y^2+z^2+2\left(xy+yz+xz\right)\)

Do \(xy+yz+xz=0\)

\(\Rightarrow x^2+y^2+z^2=x^2+y^2+z^2\) ( đpcm )

b)

\(\frac{1}{\left(a-b\right)^2}+\frac{1}{\left(b-c\right)^2}+\frac{1}{\left(c-a\right)^2}=\left(\frac{1}{a-b}+\frac{1}{b-c}+\frac{1}{c-a}\right)^2\)

\(\frac{1}{\left(a-b\right)^2}+\frac{1}{\left(b-c\right)^2}+\frac{1}{\left(c-a\right)^2}=\frac{1}{\left(a-b\right)^2}+\frac{1}{\left(b-c\right)^2}+\frac{1}{\left(c-a\right)^2}+\frac{2}{\left(a-b\right)\left(b-c\right)}+\frac{2}{\left(b-c\right)\left(c-a\right)}+\frac{2}{\left(a-b\right)\left(c-a\right)}\)

\(\Rightarrow\frac{2}{\left(a-b\right)\left(b-c\right)}+\frac{2}{\left(b-c\right)\left(c-a\right)}+\frac{2}{\left(a-b\right)\left(c-a\right)}=0\)

\(\Rightarrow2\left(\frac{1}{\left(a-b\right)\left(b-c\right)}+\frac{1}{\left(b-c\right)\left(c-a\right)}+\frac{1}{\left(a-b\right)\left(c-a\right)}\right)=0\)

\(\Rightarrow\frac{1}{\left(a-b\right)\left(b-c\right)}+\frac{1}{\left(b-c\right)\left(c-a\right)}+\frac{1}{\left(a-b\right)\left(c-a\right)}=0\)

\(\Rightarrow\frac{\left(c-a\right)^2\left(b-c\right)\left(a-b\right)+\left(a-b\right)^2\left(b-c\right)\left(c-a\right)+\left(b-c\right)^2\left(a-b\right)\left(c-a\right)}{\left(a-b\right)^2\left(b-c\right)^2\left(c-a\right)^2}=0\)

\(\Rightarrow\frac{\left(c-a\right)\left(b-c\right)\left(a-b\right)\left[\left(a-b\right)+\left(b-c\right)+\left(c-a\right)\right]}{\left(a-b\right)^2\left(b-c\right)^2\left(c-a\right)^2}=0\)

\(\Rightarrow\frac{\left(c-a\right)\left(b-c\right)\left(a-b\right)\left[\left(-a+a\right)+\left(-b+b\right)+\left(-c+c\right)\right]}{\left(a-b\right)^2\left(b-c\right)^2\left(c-a\right)^2}=0\)

\(\Rightarrow\frac{\left(c-a\right)\left(b-c\right)\left(a-b\right).0}{\left(a-b\right)^2\left(b-c\right)^2\left(c-a\right)^2}=0\)

\(\Rightarrow0=0\) ( đpcm )